Does this property of comaximal ideals always holds?Question on Comaximal IdealsUnital commutative ring and distinct maximal ideals.Where does the proof for commutative rings break down in the non-commutative ring when showing only two ideals implies the ring is a field?Direct-Sum Decomposition of an Artinian moduleProve that $m_1m_2ldots m_r=n_1n_2ldots n_s$ implies $r=s$ for distinct maximal idealsQuestion about maximal ideals in a commutative Artinian ringA property of associated prime idealsThe meaning of idempotents corresponding the standard basis in direct product of fieldsAre non-coprime ideals always contained in some prime ideal?Product of ideals equals intersection but they are not comaximal

Does this AnyDice function accurately calculate the number of ogres you make unconcious with three 4th-level castings of Sleep?

Be in awe of my brilliance!

How do I hide Chekhov's Gun?

Rules about breaking the rules. How do I do it well?

Unreachable code, but reachable with exception

Instead of Universal Basic Income, why not Universal Basic NEEDS?

Why would a flight no longer considered airworthy be redirected like this?

What are the possible solutions of the given equation?

Is it possible to upcast ritual spells?

Welcoming 2019 Pi day: How to draw the letter π?

Is having access to past exams cheating and, if yes, could it be proven just by a good grade?

How do I interpret this "sky cover" chart?

Calculus II Professor will not accept my correct integral evaluation that uses a different method, should I bring this up further?

Theorems like the Lovász Local Lemma?

PTIJ: Who should pay for Uber rides: the child or the parent?

Russian cases: A few examples, I'm really confused

Should we release the security issues we found in our product as CVE or we can just update those on weekly release notes?

What is this large pipe coming out of my roof?

How to deal with taxi scam when on vacation?

My adviser wants to be the first author

Why did it take so long to abandon sail after steamships were demonstrated?

Current sense amp + op-amp buffer + ADC: Measuring down to 0 with single supply

Good allowance savings plan?

I need to drive a 7/16" nut but am unsure how to use the socket I bought for my screwdriver



Does this property of comaximal ideals always holds?


Question on Comaximal IdealsUnital commutative ring and distinct maximal ideals.Where does the proof for commutative rings break down in the non-commutative ring when showing only two ideals implies the ring is a field?Direct-Sum Decomposition of an Artinian moduleProve that $m_1m_2ldots m_r=n_1n_2ldots n_s$ implies $r=s$ for distinct maximal idealsQuestion about maximal ideals in a commutative Artinian ringA property of associated prime idealsThe meaning of idempotents corresponding the standard basis in direct product of fieldsAre non-coprime ideals always contained in some prime ideal?Product of ideals equals intersection but they are not comaximal













5












$begingroup$


I am reading a paper in which the following result is used but I can’t see the proof of this.




let $R$ be a commutative ring with only two maximal ideals say $M_1$ and $M_2$. Suppose $m_1 in M_1$ be such that $m_1 notin M_2$ then can be always find $m_2 in M_2$ such that $m_1+m_2=1$




Any ideas?










share|cite|improve this question









$endgroup$











  • $begingroup$
    Consider the ideal generated by $M_2$ and $m_1$, this ideal must be $R=(1)$ since $M_2$ is maximal
    $endgroup$
    – B.Swan
    2 hours ago










  • $begingroup$
    @B.Swan this approach doesn't work, to see why try writing out the details
    $endgroup$
    – Alex Mathers
    2 hours ago






  • 1




    $begingroup$
    Set $I=(M_2 cup m_1) $, the ideal generated by $M_2$ and $m_1$. Elements of $I$ have the form $x+rm_1$, where $x in M_2$ and $r in R$. Since $m_1 notin M_2$ and $M_2$ maximal, it follows $I=R$. Thus there exists $s in R$ with $1=x+sm_1$. And I guess one gets stuck here. Sorry for the wrong approach and thanks for pointing it out.
    $endgroup$
    – B.Swan
    2 hours ago
















5












$begingroup$


I am reading a paper in which the following result is used but I can’t see the proof of this.




let $R$ be a commutative ring with only two maximal ideals say $M_1$ and $M_2$. Suppose $m_1 in M_1$ be such that $m_1 notin M_2$ then can be always find $m_2 in M_2$ such that $m_1+m_2=1$




Any ideas?










share|cite|improve this question









$endgroup$











  • $begingroup$
    Consider the ideal generated by $M_2$ and $m_1$, this ideal must be $R=(1)$ since $M_2$ is maximal
    $endgroup$
    – B.Swan
    2 hours ago










  • $begingroup$
    @B.Swan this approach doesn't work, to see why try writing out the details
    $endgroup$
    – Alex Mathers
    2 hours ago






  • 1




    $begingroup$
    Set $I=(M_2 cup m_1) $, the ideal generated by $M_2$ and $m_1$. Elements of $I$ have the form $x+rm_1$, where $x in M_2$ and $r in R$. Since $m_1 notin M_2$ and $M_2$ maximal, it follows $I=R$. Thus there exists $s in R$ with $1=x+sm_1$. And I guess one gets stuck here. Sorry for the wrong approach and thanks for pointing it out.
    $endgroup$
    – B.Swan
    2 hours ago














5












5








5


1



$begingroup$


I am reading a paper in which the following result is used but I can’t see the proof of this.




let $R$ be a commutative ring with only two maximal ideals say $M_1$ and $M_2$. Suppose $m_1 in M_1$ be such that $m_1 notin M_2$ then can be always find $m_2 in M_2$ such that $m_1+m_2=1$




Any ideas?










share|cite|improve this question









$endgroup$




I am reading a paper in which the following result is used but I can’t see the proof of this.




let $R$ be a commutative ring with only two maximal ideals say $M_1$ and $M_2$. Suppose $m_1 in M_1$ be such that $m_1 notin M_2$ then can be always find $m_2 in M_2$ such that $m_1+m_2=1$




Any ideas?







abstract-algebra ring-theory commutative-algebra maximal-and-prime-ideals






share|cite|improve this question













share|cite|improve this question











share|cite|improve this question




share|cite|improve this question










asked 2 hours ago









Math LoverMath Lover

1,029315




1,029315











  • $begingroup$
    Consider the ideal generated by $M_2$ and $m_1$, this ideal must be $R=(1)$ since $M_2$ is maximal
    $endgroup$
    – B.Swan
    2 hours ago










  • $begingroup$
    @B.Swan this approach doesn't work, to see why try writing out the details
    $endgroup$
    – Alex Mathers
    2 hours ago






  • 1




    $begingroup$
    Set $I=(M_2 cup m_1) $, the ideal generated by $M_2$ and $m_1$. Elements of $I$ have the form $x+rm_1$, where $x in M_2$ and $r in R$. Since $m_1 notin M_2$ and $M_2$ maximal, it follows $I=R$. Thus there exists $s in R$ with $1=x+sm_1$. And I guess one gets stuck here. Sorry for the wrong approach and thanks for pointing it out.
    $endgroup$
    – B.Swan
    2 hours ago

















  • $begingroup$
    Consider the ideal generated by $M_2$ and $m_1$, this ideal must be $R=(1)$ since $M_2$ is maximal
    $endgroup$
    – B.Swan
    2 hours ago










  • $begingroup$
    @B.Swan this approach doesn't work, to see why try writing out the details
    $endgroup$
    – Alex Mathers
    2 hours ago






  • 1




    $begingroup$
    Set $I=(M_2 cup m_1) $, the ideal generated by $M_2$ and $m_1$. Elements of $I$ have the form $x+rm_1$, where $x in M_2$ and $r in R$. Since $m_1 notin M_2$ and $M_2$ maximal, it follows $I=R$. Thus there exists $s in R$ with $1=x+sm_1$. And I guess one gets stuck here. Sorry for the wrong approach and thanks for pointing it out.
    $endgroup$
    – B.Swan
    2 hours ago
















$begingroup$
Consider the ideal generated by $M_2$ and $m_1$, this ideal must be $R=(1)$ since $M_2$ is maximal
$endgroup$
– B.Swan
2 hours ago




$begingroup$
Consider the ideal generated by $M_2$ and $m_1$, this ideal must be $R=(1)$ since $M_2$ is maximal
$endgroup$
– B.Swan
2 hours ago












$begingroup$
@B.Swan this approach doesn't work, to see why try writing out the details
$endgroup$
– Alex Mathers
2 hours ago




$begingroup$
@B.Swan this approach doesn't work, to see why try writing out the details
$endgroup$
– Alex Mathers
2 hours ago




1




1




$begingroup$
Set $I=(M_2 cup m_1) $, the ideal generated by $M_2$ and $m_1$. Elements of $I$ have the form $x+rm_1$, where $x in M_2$ and $r in R$. Since $m_1 notin M_2$ and $M_2$ maximal, it follows $I=R$. Thus there exists $s in R$ with $1=x+sm_1$. And I guess one gets stuck here. Sorry for the wrong approach and thanks for pointing it out.
$endgroup$
– B.Swan
2 hours ago





$begingroup$
Set $I=(M_2 cup m_1) $, the ideal generated by $M_2$ and $m_1$. Elements of $I$ have the form $x+rm_1$, where $x in M_2$ and $r in R$. Since $m_1 notin M_2$ and $M_2$ maximal, it follows $I=R$. Thus there exists $s in R$ with $1=x+sm_1$. And I guess one gets stuck here. Sorry for the wrong approach and thanks for pointing it out.
$endgroup$
– B.Swan
2 hours ago











2 Answers
2






active

oldest

votes


















5












$begingroup$

First notice that $1-m_1$ cannot be a unit, because this would imply $m_1$ is in the Jacobson radical of $R$, and in particular we would have $m_1in M_2$.



Now it follows that the ideal of $R$ generated by $1-m_1$ must be contained in a maximal ideal, but it cannot be contained in $M_1$ because then it would follow that $1in M_1$. Thus this ideal is contained in $M_2$ (the only other maximal ideal), i.e. you get $1-m_1in M_2$.




Edit: I think my reasoning for $1-m_1$ not being a unit is wrong (it seems we would need that $1-m_1x$ is a unit for every $xin R$ to conclude $m_1$ is in the Jacobson radical). The rest of the argument goes through, so I'm going to leave my answer up for a while in hopes that somebody can help figure that part out.






share|cite|improve this answer











$endgroup$




















    3












    $begingroup$

    Take $R=mathbbQtimesmathbbQ$, $M_1=mathbbQtimes0$, $M_2=0timesmathbbQ$, and $m_1=(2,0)in M_1setminus M_2$. Then $(1,1)inmathbbQtimesmathbbQ$ satisfies that $$(1,1)-(2,0)=(-1,1)notin M_2$$



    Therefore, that property is not satisfied in general.



    Maybe the property that they are really using is that there exist $ain M_1$ and $bin M_2$ such that $a+b=1$. Not arbitrary $a,b$. This other property is immediate by using the maximality of $M_1$ and $M_2$, which implies that $M_1+M_2=R$.






    share|cite|improve this answer











    $endgroup$












      Your Answer





      StackExchange.ifUsing("editor", function ()
      return StackExchange.using("mathjaxEditing", function ()
      StackExchange.MarkdownEditor.creationCallbacks.add(function (editor, postfix)
      StackExchange.mathjaxEditing.prepareWmdForMathJax(editor, postfix, [["$", "$"], ["\\(","\\)"]]);
      );
      );
      , "mathjax-editing");

      StackExchange.ready(function()
      var channelOptions =
      tags: "".split(" "),
      id: "69"
      ;
      initTagRenderer("".split(" "), "".split(" "), channelOptions);

      StackExchange.using("externalEditor", function()
      // Have to fire editor after snippets, if snippets enabled
      if (StackExchange.settings.snippets.snippetsEnabled)
      StackExchange.using("snippets", function()
      createEditor();
      );

      else
      createEditor();

      );

      function createEditor()
      StackExchange.prepareEditor(
      heartbeatType: 'answer',
      autoActivateHeartbeat: false,
      convertImagesToLinks: true,
      noModals: true,
      showLowRepImageUploadWarning: true,
      reputationToPostImages: 10,
      bindNavPrevention: true,
      postfix: "",
      imageUploader:
      brandingHtml: "Powered by u003ca class="icon-imgur-white" href="https://imgur.com/"u003eu003c/au003e",
      contentPolicyHtml: "User contributions licensed under u003ca href="https://creativecommons.org/licenses/by-sa/3.0/"u003ecc by-sa 3.0 with attribution requiredu003c/au003e u003ca href="https://stackoverflow.com/legal/content-policy"u003e(content policy)u003c/au003e",
      allowUrls: true
      ,
      noCode: true, onDemand: true,
      discardSelector: ".discard-answer"
      ,immediatelyShowMarkdownHelp:true
      );



      );













      draft saved

      draft discarded


















      StackExchange.ready(
      function ()
      StackExchange.openid.initPostLogin('.new-post-login', 'https%3a%2f%2fmath.stackexchange.com%2fquestions%2f3148803%2fdoes-this-property-of-comaximal-ideals-always-holds%23new-answer', 'question_page');

      );

      Post as a guest















      Required, but never shown

























      2 Answers
      2






      active

      oldest

      votes








      2 Answers
      2






      active

      oldest

      votes









      active

      oldest

      votes






      active

      oldest

      votes









      5












      $begingroup$

      First notice that $1-m_1$ cannot be a unit, because this would imply $m_1$ is in the Jacobson radical of $R$, and in particular we would have $m_1in M_2$.



      Now it follows that the ideal of $R$ generated by $1-m_1$ must be contained in a maximal ideal, but it cannot be contained in $M_1$ because then it would follow that $1in M_1$. Thus this ideal is contained in $M_2$ (the only other maximal ideal), i.e. you get $1-m_1in M_2$.




      Edit: I think my reasoning for $1-m_1$ not being a unit is wrong (it seems we would need that $1-m_1x$ is a unit for every $xin R$ to conclude $m_1$ is in the Jacobson radical). The rest of the argument goes through, so I'm going to leave my answer up for a while in hopes that somebody can help figure that part out.






      share|cite|improve this answer











      $endgroup$

















        5












        $begingroup$

        First notice that $1-m_1$ cannot be a unit, because this would imply $m_1$ is in the Jacobson radical of $R$, and in particular we would have $m_1in M_2$.



        Now it follows that the ideal of $R$ generated by $1-m_1$ must be contained in a maximal ideal, but it cannot be contained in $M_1$ because then it would follow that $1in M_1$. Thus this ideal is contained in $M_2$ (the only other maximal ideal), i.e. you get $1-m_1in M_2$.




        Edit: I think my reasoning for $1-m_1$ not being a unit is wrong (it seems we would need that $1-m_1x$ is a unit for every $xin R$ to conclude $m_1$ is in the Jacobson radical). The rest of the argument goes through, so I'm going to leave my answer up for a while in hopes that somebody can help figure that part out.






        share|cite|improve this answer











        $endgroup$















          5












          5








          5





          $begingroup$

          First notice that $1-m_1$ cannot be a unit, because this would imply $m_1$ is in the Jacobson radical of $R$, and in particular we would have $m_1in M_2$.



          Now it follows that the ideal of $R$ generated by $1-m_1$ must be contained in a maximal ideal, but it cannot be contained in $M_1$ because then it would follow that $1in M_1$. Thus this ideal is contained in $M_2$ (the only other maximal ideal), i.e. you get $1-m_1in M_2$.




          Edit: I think my reasoning for $1-m_1$ not being a unit is wrong (it seems we would need that $1-m_1x$ is a unit for every $xin R$ to conclude $m_1$ is in the Jacobson radical). The rest of the argument goes through, so I'm going to leave my answer up for a while in hopes that somebody can help figure that part out.






          share|cite|improve this answer











          $endgroup$



          First notice that $1-m_1$ cannot be a unit, because this would imply $m_1$ is in the Jacobson radical of $R$, and in particular we would have $m_1in M_2$.



          Now it follows that the ideal of $R$ generated by $1-m_1$ must be contained in a maximal ideal, but it cannot be contained in $M_1$ because then it would follow that $1in M_1$. Thus this ideal is contained in $M_2$ (the only other maximal ideal), i.e. you get $1-m_1in M_2$.




          Edit: I think my reasoning for $1-m_1$ not being a unit is wrong (it seems we would need that $1-m_1x$ is a unit for every $xin R$ to conclude $m_1$ is in the Jacobson radical). The rest of the argument goes through, so I'm going to leave my answer up for a while in hopes that somebody can help figure that part out.







          share|cite|improve this answer














          share|cite|improve this answer



          share|cite|improve this answer








          edited 1 hour ago

























          answered 2 hours ago









          Alex MathersAlex Mathers

          11.1k21344




          11.1k21344





















              3












              $begingroup$

              Take $R=mathbbQtimesmathbbQ$, $M_1=mathbbQtimes0$, $M_2=0timesmathbbQ$, and $m_1=(2,0)in M_1setminus M_2$. Then $(1,1)inmathbbQtimesmathbbQ$ satisfies that $$(1,1)-(2,0)=(-1,1)notin M_2$$



              Therefore, that property is not satisfied in general.



              Maybe the property that they are really using is that there exist $ain M_1$ and $bin M_2$ such that $a+b=1$. Not arbitrary $a,b$. This other property is immediate by using the maximality of $M_1$ and $M_2$, which implies that $M_1+M_2=R$.






              share|cite|improve this answer











              $endgroup$

















                3












                $begingroup$

                Take $R=mathbbQtimesmathbbQ$, $M_1=mathbbQtimes0$, $M_2=0timesmathbbQ$, and $m_1=(2,0)in M_1setminus M_2$. Then $(1,1)inmathbbQtimesmathbbQ$ satisfies that $$(1,1)-(2,0)=(-1,1)notin M_2$$



                Therefore, that property is not satisfied in general.



                Maybe the property that they are really using is that there exist $ain M_1$ and $bin M_2$ such that $a+b=1$. Not arbitrary $a,b$. This other property is immediate by using the maximality of $M_1$ and $M_2$, which implies that $M_1+M_2=R$.






                share|cite|improve this answer











                $endgroup$















                  3












                  3








                  3





                  $begingroup$

                  Take $R=mathbbQtimesmathbbQ$, $M_1=mathbbQtimes0$, $M_2=0timesmathbbQ$, and $m_1=(2,0)in M_1setminus M_2$. Then $(1,1)inmathbbQtimesmathbbQ$ satisfies that $$(1,1)-(2,0)=(-1,1)notin M_2$$



                  Therefore, that property is not satisfied in general.



                  Maybe the property that they are really using is that there exist $ain M_1$ and $bin M_2$ such that $a+b=1$. Not arbitrary $a,b$. This other property is immediate by using the maximality of $M_1$ and $M_2$, which implies that $M_1+M_2=R$.






                  share|cite|improve this answer











                  $endgroup$



                  Take $R=mathbbQtimesmathbbQ$, $M_1=mathbbQtimes0$, $M_2=0timesmathbbQ$, and $m_1=(2,0)in M_1setminus M_2$. Then $(1,1)inmathbbQtimesmathbbQ$ satisfies that $$(1,1)-(2,0)=(-1,1)notin M_2$$



                  Therefore, that property is not satisfied in general.



                  Maybe the property that they are really using is that there exist $ain M_1$ and $bin M_2$ such that $a+b=1$. Not arbitrary $a,b$. This other property is immediate by using the maximality of $M_1$ and $M_2$, which implies that $M_1+M_2=R$.







                  share|cite|improve this answer














                  share|cite|improve this answer



                  share|cite|improve this answer








                  edited 36 mins ago

























                  answered 1 hour ago









                  user647486user647486

                  513




                  513



























                      draft saved

                      draft discarded
















































                      Thanks for contributing an answer to Mathematics Stack Exchange!


                      • Please be sure to answer the question. Provide details and share your research!

                      But avoid


                      • Asking for help, clarification, or responding to other answers.

                      • Making statements based on opinion; back them up with references or personal experience.

                      Use MathJax to format equations. MathJax reference.


                      To learn more, see our tips on writing great answers.




                      draft saved


                      draft discarded














                      StackExchange.ready(
                      function ()
                      StackExchange.openid.initPostLogin('.new-post-login', 'https%3a%2f%2fmath.stackexchange.com%2fquestions%2f3148803%2fdoes-this-property-of-comaximal-ideals-always-holds%23new-answer', 'question_page');

                      );

                      Post as a guest















                      Required, but never shown





















































                      Required, but never shown














                      Required, but never shown












                      Required, but never shown







                      Required, but never shown

































                      Required, but never shown














                      Required, but never shown












                      Required, but never shown







                      Required, but never shown







                      Popular posts from this blog

                      कुँवर स्रोत दिक्चालन सूची"कुँवर""राणा कुँवरके वंशावली"

                      Why is a white electrical wire connected to 2 black wires?How to wire a light fixture with 3 white wires in box?How should I wire a ceiling fan when there's only three wires in the box?Two white, two black, two ground, and red wire in ceiling box connected to switchWhy is there a white wire connected to multiple black wires in my light box?How to wire a light with two white wires and one black wireReplace light switch connected to a power outlet with dimmer - two black wires to one black and redHow to wire a light with multiple black/white/green wires from the ceiling?Ceiling box has 2 black and white wires but fan/ light only has 1 of eachWhy neutral wire connected to load wire?Switch with 2 black, 2 white, 2 ground and 1 red wire connected to ceiling light and a receptacle?

                      चैत्य भूमि चित्र दीर्घा सन्दर्भ बाहरी कडियाँ दिक्चालन सूची"Chaitya Bhoomi""Chaitya Bhoomi: Statue of Equality in India""Dadar Chaitya Bhoomi: Statue of Equality in India""Ambedkar memorial: Centre okays transfer of Indu Mill land"चैत्यभमि